Decomposing and Recomposing

We have a partition of 2018. If the maximum value of the product of the numbers in the partition can be expressed as a × b c , a \times b^c, where a a and b b are primes and c c is an integer, then what is a + b + c ? a+b+c?


The answer is 677.

This section requires Javascript.
You are seeing this because something didn't load right. We suggest you, (a) try refreshing the page, (b) enabling javascript if it is disabled on your browser and, finally, (c) loading the non-javascript version of this page . We're sorry about the hassle.

4 solutions

Romain Bouchard
Feb 23, 2018

Let's call a partition of any integer n n maximal if the product of the elements of the partition is maximal. (e.g. for n = 4 n=4 we have the three following partitions : { 1 , 1 , 1 , 1 } \{1,1,1,1\} , { 1 , 1 , 2 } \{1,1,2\} , { 1 , 3 } \{1,3\} as well as two maximal partitions : { 2 , 2 } \{2,2\} and { 4 } \{4\} .)

  • It is obvious that 1 1 cannot be an element of a maximal partition because we could then get rid of it by adding 1 1 to another element, therefore keeping the sum intact and inscreasing the value of the product.

  • In the same way, we can discard any partition with an element p 4 p \geq 4 because then we could split it into p 2 p-2 and 2 2 therefore keeping the sum intact and substituting the corresponding factor in the product value from p p to 2 ( p 2 ) 2(p-2) which is always p \geq p . We now just proved that there is a maximal partition with only 2 2 s and 3 3 s.

  • We can then show that no more than two 2 2 can be elements of the maximal partition : otherwise we could replace 2 + 2 + 2 2+2+2 by 3 + 3 3+3 therfore keeping the sum intact while increasing a factor in the product from 2 × 2 × 2 2\times 2\times 2 to 3 × 3 3 \times 3 .

  • Since 2018 = 3 × 672 + 2 2018 = 3\times 672 +2 , we get a maximum value for the product at 2 3 672 2\cdot 3^{672} . Hence a + b + c = 2 + 3 + 672 = 677 a+b+c = 2+3+672 = 677 .

Isn't the second last line is 2 × 3 672 2×3^{672} .

Chan Tin Ping - 3 years, 3 months ago

Log in to reply

Yes, typo :( Thanks

Romain Bouchard - 3 years, 3 months ago

Simple and elegant. I would simply add that x y > y x x^y>y^x iff l n ( x ) / x > l n ( y ) / y ln(x)/x>ln(y)/y . The function x l n ( x ) / x x\mapsto ln(x)/x has maximal value at e 2.718 e\approx 2.718 thus to get a so-called maximal partition, one should multiply only three's and two's, with preference for the three's.

Maxence Seymat - 3 years, 3 months ago

Log in to reply

That's a nice argument indeed.

Romain Bouchard - 3 years, 3 months ago

Why AM-GM can not be used .Using this I am getting answer 2 1009 2^{1009}

Kushal Bose - 3 years, 3 months ago

Log in to reply

I agree that with the multiset { 2 , 2 , 2 , . . . , 2 } \{2,2,2,...,2\} you have a product value equal to 2 1009 2^{1009} but we are looking for the maximum product value and as you can see 2 1009 < 2. 3 672 2^{1009} < 2.3^{672} .

Romain Bouchard - 3 years, 3 months ago

Technically, what you mean is multiset , not set, since elements of a set are all distinct.

Denis Husadzic - 3 years, 3 months ago

Log in to reply

I went with the intuitive idea of a set. Thanks for the input.

Romain Bouchard - 3 years, 3 months ago

Why can't we just set "a" to whatever the final product is, "b" to 1 and "c" to any number we want? Is it just because there isn't an upper limit on "c" and thus it can't be maximised?

Thane Symens - 3 years, 3 months ago

Log in to reply

Interesting point, maybe the solution should be clarified by adding b>1, b!=1 or something like that... Althought I think is in fact clear that the requered solution is the one with b=3. You can't put infinit on the solition and I think you are write about the lack of upper limit.

Pau Cantos - 3 years, 3 months ago

Log in to reply

Thanks! I've edited the problem to clarify.

Andrew Hayes Staff - 3 years, 3 months ago

Brilliant observation

vikas srivastava - 3 years, 3 months ago

Couldn't understand

Mr. India - 3 years, 3 months ago

Could u pls explain why did u break 2018 as 3*672+2??

erica phillips - 3 years, 3 months ago

Log in to reply

This took me a second as well, as I am not used to partitions. The product of the numbers that make up the partition (which consist only of numbers a, b, c). So a b^c means that the number "a" appears once in the partition, whereas the number "b" appears "c" times. The definition of a partition is the total number of unique ways to sum of all numbers equal to a certain value (in this case 2018). So 2018 must be equal to that 1 a + (b +b +b +b... +b) c times, i.e. 2018 = a + c b. Hope this helps. I think Daniel Xiang's solution below is much less hand wavy (though the reasoning in Romain's is nice). Still requires this understanding though.

B M - 3 years, 3 months ago

Log in to reply

Yes basically of all the ways that you can break down 2018 2018 (i.e. from 1 + 1 + 1 + . . . + 1 1+1+1+...+1 to 2018 2018 , the one for which the product of all the elements is maximal is 3 + 3 + 3 + 3... + 3 + 2 3+3+3+3...+3+2 i.e. 3 × 672 + 2 3 \times 672 + 2 .

Romain Bouchard - 3 years, 3 months ago

Thanks for the explanation but I actually had a problem that how to figure out the no. 672,pls help me out there!!!thanks in advance

erica phillips - 3 years, 3 months ago

Log in to reply

@Erica Phillips I would seek out the method of Lagrange multipliers as done in Daniel Xiang's solution. This method sets the equation you are trying to maximize f(x,y,z,...) given a constraint C(x,y,z,...)

L(x,y,z,...; lambda) = f(x,y,z,...) + lambda*C(x,y,z,...)

and then find extreme values given by setting the partial derivatives of each variable to zero. You will have a system of equations which can be used to deduce information on each of the variables. (Having issues accessing the math palette for text). This of course can be generalized for multiple constraints but is not needed here.

B M - 3 years, 2 months ago

Log in to reply

@B M Thanks a lot!!!

erica phillips - 3 years, 2 months ago
Daniel Xiang
Mar 6, 2018

Since the product of the integers can be expressed as a × b c a\times b^c we deduce that a + b c = 2018 a+bc=2018

To maximise a × b c a \times b^c we introduce the auxiliary function

L ( a , b , c , λ ) : = a b c λ ( a + b c 2018 ) \mathcal{L}(a, b, c, \lambda) := ab^c - \lambda (a+bc-2018)

It follows that

L a = L b = L c = 0 \displaystyle \frac{\partial \mathcal{L} }{\partial a}= \frac{\partial \mathcal{L} }{\partial b}= \frac{\partial \mathcal{L} }{\partial c}=0

Solving the equations,

L a = b c λ = 0 L b = a c b c 1 c λ = 0 L c = a b c ln b b λ = 0 \begin{aligned}&\frac{\partial \mathcal{L} }{\partial a} = b^c - \lambda = 0 \\ &\frac{\partial \mathcal{L} }{\partial b} = acb^{c-1} - c\lambda =0 \\ &\frac{\partial \mathcal{L} }{\partial c} = ab^c\ln b - b\lambda = 0\end{aligned}

we have

b c = a b c 1 = a b c 1 ln b b^c = ab^{c-1}=ab^{c-1}\ln b

Since a , b , c Z + a, b, c \in \mathbb{Z^{+}} , it is clear that a b e = 2.718 a\approx b \approx e=2.718\dots

Which implies that the possible values for a , b a, b should be 2 2 or 3 3

if a = 3 a = 3 and b = 2 b=2 then c = 2018 3 2 = 1007.5 Z \displaystyle c = \frac{2018-3}{2} = 1007.5 \notin \mathbb{Z}

if a = b = 3 a = b=3 then c = 2018 3 3 = 671.66 Z \displaystyle c = \frac{2018-3}{3} = 671.66\dots \notin \mathbb{Z}

if a = 2 a = 2 and b = 3 b = 3 then c = 2018 2 3 = 672 Z \displaystyle c = \frac{2018-2}{3} = 672 \in \mathbb{Z}

if a = b = 2 a=b=2 then c = 2018 2 2 = 1008 Z c = \displaystyle \frac{2018-2}{2} = 1008 \in \mathbb{Z}

Since 2 1008 < 3 672 2^{1008} < 3^{672} we deduce that ( a , b , c ) = ( 2 , 3 , 672 ) (a, b, c) = (2, 3, 672) is the correct answer

Finally a + b + c = 2 + 3 + 672 = 677 a+b+c=2+3+672=\boxed{677}

I love it.

Ròtêm Assouline - 3 years, 2 months ago
Meneghin Mauro
Mar 6, 2018

Another way to see that factoring with 3 as much as possible leads to maximal product.

Considering the function x n / x x^{n/x} , this has a maximum x = e x=e , the closest integer 3 happens to give the maximum for integers.

So 2018 = 3 672 + 2 2018=3*672+2 leads to 2 3 672 2*3^{672}

Stewart Gordon
Mar 10, 2018

"We now just proved that only 2 2 and 3 3 can be elements of a maximal partition." - this isn't true. For instance, { 4 } \{| 4 |\} is a maximal partition of 4 4 . (But what can be said is that we can disregard any partition containing a 4 4 , since if such a partition is maximal then there is also a maximal partition without any 4 4 s in.)

Agreed. I changed the wording.

Romain Bouchard - 3 years, 3 months ago

0 pending reports

×

Problem Loading...

Note Loading...

Set Loading...